Search found 3161 matches


5 boys and 4 girls are going for a movie. Andy and Sally are part of this group. For a restriction that no two boys or girls can sit beside each other, what is the probability of Andy sitting besides Sally? OA-2/5 Many probability questions can be solved intuitively rather than with long calculatio...

by Stuart@KaplanGMAT

Tue Dec 31, 2013 12:21 am
Forum: Problem Solving
Topic: Probability
Replies: 4
Views: 2333

One more approach! The word "ancient" is a present use word - in other words, something is ancient from today's perspective. Accordingly, if we're talking about a present discovery, we don't say "what appeared to be an ancient hunting ground"; instead, we say "what appears t...

by Stuart@KaplanGMAT

Sun Dec 29, 2013 12:53 am
Forum: Sentence Correction
Topic: Researchers
Replies: 3
Views: 2055

A test is taken by 100 people and possible scores are the integers between 0 and 50, inclusive. Would the avg. score be greater than 30? (Answer Yes, No or Uncertain) Condition- No more than 2 test takers scored any given score. Ans: NO This one is from MGMAT's Advanced Quant book. I did not unders...

by Stuart@KaplanGMAT

Sun Dec 29, 2013 12:45 am
Forum: Problem Solving
Topic: MGMAT Advanced Quant- Visual Ques.
Replies: 2
Views: 1913

Hi Sid, you'll find some decent free resources on the net, you just have to be really careful. I'd always look for reviews of the source before diving in and using its questions. For all you know, those free questions were written by some bored guy living in his mom's basement! Depending on how much...

by Stuart@KaplanGMAT

Sat Dec 28, 2013 6:43 pm
Forum: Critical Reasoning
Topic: Skeletal Heat
Replies: 5
Views: 2993

A medical researcher must choose one of 14 patients to receive an experimental medicine called Progaine. The researcher must then choose one of the remaining 13 patients to receive another medicine, called Ropecia. Finally, the researcher administers a placebo to one of the remaining 12 patients. A...

by Stuart@KaplanGMAT

Sat Dec 28, 2013 8:46 am
Forum: Problem Solving
Topic: Combinatorics: Solution Explanation
Replies: 4
Views: 1824

Samuel is obviously a bad fisherman. During the past season, in which he and the five members of his team spent four months on a boat together off Dutch Harbor, AK, he caught fewer fish than any of his teammates. Which of the following, if true, most weakens the argument above? A) Two seasons ago, ...

by Stuart@KaplanGMAT

Sat Dec 28, 2013 8:35 am
Forum: Critical Reasoning
Topic: Samuel's Fishing Dilemma
Replies: 2
Views: 1537

An advertisement for E-News, a subscription-only online newspaper with no ads accompanying its content, argued that individuals should subscribe to E-News so as to eliminate wasted time that results from seeing and ignoring advertisements while attempting to read newspapers that feature ads. Which ...

by Stuart@KaplanGMAT

Sat Dec 28, 2013 8:29 am
Forum: Critical Reasoning
Topic: E-News Ads
Replies: 1
Views: 1528

On a recent expedition to a remote region of northern Canada, scientists uncovered skeletal remains from about 100,000 years ago. Surprisingly, all the skeletal remains, which included many species from differing biological families and spanned about two thousand years, showed evidence of experienc...

by Stuart@KaplanGMAT

Sat Dec 28, 2013 8:22 am
Forum: Critical Reasoning
Topic: Skeletal Heat
Replies: 5
Views: 2993

C) When a number is not repeated (1,2,3) (6,5,4) etc = 6!/(3!*3!) = 5 * 4 = 20 Now this would be 6Choose3. As you are choosing 3 different numbers from choice of 6 possible numbers. Shouldn't the above be simply--- 6x5x4= 120? Why 6C3? Please explain. Thanks. Hi Nipun! The problem with treating thi...

by Stuart@KaplanGMAT

Sat Dec 28, 2013 8:16 am
Forum: Problem Solving
Topic: Dice
Replies: 9
Views: 7235

Hi everyone. I have exactly the same question! Can someone please explain the rationale behind the Flashcards formula : AB + BA ----- CDC I've noticed similar inconsistencies (83+38 = 121 -> ok ; but 48+84 = 132 -> not ok)and would like to understand the rules / exceptions behind the above formula....

by Stuart@KaplanGMAT

Sat Dec 28, 2013 2:59 am
Forum: Problem Solving
Topic: Quicker + alternate methods
Replies: 7
Views: 2592

But 4/8=1/2=0.5 .Apparently,its tenth digit is 0 Hi! I'm assuming that this last part in quotes is your actually comment/question. It's important to know the place names for the various digits in a numbers, so let's do a quick review, using the example of: 12345.6789 We start numbering to the left ...

by Stuart@KaplanGMAT

Sat Dec 28, 2013 1:22 am
Forum: Data Sufficiency
Topic: tenth digit in decimal representation
Replies: 11
Views: 61917

The conclusion of this argument is : Therefore, the number of vehicles manufactured and sold in Esteria must have decreased in the last five years. Answer choice E addresses only the "manufacture vehicles but not sold. I understand that no response addresses these two issues simultaneously. Ho...

by Stuart@KaplanGMAT

Fri Dec 27, 2013 7:20 pm
Forum: Critical Reasoning
Topic: MGMAT CAT Esteria vs Burdistan
Replies: 11
Views: 4155

In the month of August, Pentheus Corporation made $200,000 in profit. Pentheus made 6% of that profit on the second Wednesday of August. If the profits that day were approximately 14.5% of the revenue for that day, then what was Pentheus’s revenue on the second Wednesday of August? (A) $65,536 (B...


Nobody knows exactly how many languages there are in the world, partly because of the difficulty of distinguishing between a language and the sub-languages or dialects within it , but those who have tried to count typically have found about five thousand. OA A Is usage of within it correct??? Hi! I...

by Stuart@KaplanGMAT

Thu Dec 26, 2013 2:45 am
Forum: Sentence Correction
Topic: Lanuguages
Replies: 1
Views: 1350

Hello, Can you please assist with this: If three glasses contain 1/6 liter, 1/8 liter, and 1/9 liter of water, respectively, then the total amount of water in the three glasses is between… (A) 0 and 1/3 liters (B) 1/3 and 1/2 liters (C) 1/2 and 2/3 liters (D) 2/3 and 5/6 liters (E) 5/6 and 1 lite...

by Stuart@KaplanGMAT

Wed Dec 25, 2013 9:24 pm
Forum: Problem Solving
Topic: To find the total amount of water in 3 glasses
Replies: 5
Views: 1831